Volume integration using washermethod

  • Thread starter Thread starter peercortsa
  • Start date Start date
  • Tags Tags
    Integration Volume
Click For Summary
The discussion focuses on using the washer method to calculate the volume of a region bounded by y = x², y = 0, x = 1, and x = 4, when rotated about the line x = -1. The integral setup involves the outer radius as 1 + √y and the inner radius as 1, with the integration bounds from 0 to 16. Participants confirm that the volume formula should be expressed as π∫(outer radius)² - (inner radius)² dx. There is confusion regarding the correct setup and evaluation of the integral, as the original poster struggles to arrive at the expected answer of 339π/2. The conversation emphasizes the need for proper integration limits and the correct formulation of the volume equation.
peercortsa
Messages
2
Reaction score
0
volume integration using "washer method"

Homework Statement


The region R bounded by y = x^{2}, y = 0, x = 1 and x = 4 is rotated about x = −1

Homework Equations


i know that these equations take on the form of \pi\int_a^b \\((outer radius)^{2} - (inner radius)^{2})\\,dx



The Attempt at a Solution


so i set the problem up like this and still can't get the correct answer which is 339\pi/2

-for the bounds i know that the integral i want to evaluate is between 0 and 16 based on the line y=0 and the function y=x^{2} evaluated at x=4.

-the outer radius is 1+\sqrt{y} and the inner radius is just 1

\pi\int_0^{16}\\ (1+\sqrt{y})^2-(1)^2\\,dy
 
Last edited:
Physics news on Phys.org
Welcome to PF!

peercortsa said:
The region R bounded by y = x^{2}, y = 0, x = 1 and x = 4 is rotated about x = −1

i know that these equations take on the form of \pi\int_a^b \\((outer radius)^{2} - (inner radius)^{2})\\,dx

Hi peercortsa! Welcome to PF! :smile:

That's right … but then your thickness is dx, so that's what you must integrate over …

∫(blah blah) dx, where blah blah is entirely a function of x. :wink:
 
so... did i even set up the equation correctly because i still can't seem to get the right answer no matter what i try :confused:
 
peercortsa said:
so... did i even set up the equation correctly because i still can't seem to get the right answer no matter what i try :confused:

Your π∫…dx is correct for the area

but you still need to put the height inside the ∫ to make the volume.

Have a go! :smile:
 
Question: A clock's minute hand has length 4 and its hour hand has length 3. What is the distance between the tips at the moment when it is increasing most rapidly?(Putnam Exam Question) Answer: Making assumption that both the hands moves at constant angular velocities, the answer is ## \sqrt{7} .## But don't you think this assumption is somewhat doubtful and wrong?

Similar threads

  • · Replies 2 ·
Replies
2
Views
2K
  • · Replies 9 ·
Replies
9
Views
2K
Replies
4
Views
2K
  • · Replies 1 ·
Replies
1
Views
2K
  • · Replies 7 ·
Replies
7
Views
2K
Replies
2
Views
1K
  • · Replies 10 ·
Replies
10
Views
2K
  • · Replies 5 ·
Replies
5
Views
2K
  • · Replies 19 ·
Replies
19
Views
2K
  • · Replies 14 ·
Replies
14
Views
2K